Question:
Comment trouver tous les moments pairs (et les moments impairs) pour $ f_X (x) = \ frac {1} {2} e ^ {- | x |} $?
Ron Snow
2019-09-17 02:49:51 UTC
view on stackexchange narkive permalink

On m'a demandé de trouver une formule pour tous les moments pairs de la forme $ E (X ^ {2n}) $ et tous les moments impairs de la forme $ E (X ^ {2n + 1}) $ en utilisant un mgf.Pouvez-vous m'aider à trouver les moments pairs?J'essaierai de résoudre les moments impairs en utilisant des techniques similaires.

Soit $ X $ une variable aléatoire continue ayant la densité $ f_X (x) = \ frac {1}{2} e ^ {- | x |} $ , où $ - \ infty < x < \ infty $ .

Mon travail: j'ai pu obtenir la fonction de génération de moment suivante: $ M_X (t) = (1-t ^ 2) ^ {- 1}. $ Cependant, je ne sais pas trop où aller à partir d'ici.Merci pour votre aide.

Astuce: Si vous pouvez exprimer $ M_X (t) $ comme une série de puissance dans $ t $, (c'est-à-dire quelque chose qui ressemble à $ 1 + m_1t + m_2t ^ 2 + m_3 t ^ 3 + \ cdots $) alors vous pouvez trouver$ E [X ^ n] $ en termes de $ m_n $
Je pense que je vois ce que vous dites.Puisque $ M_X (t) = (1-t ^ 2) ^ {- 1} $, alors cela ressemble légèrement à une série géométrique: $ (1-t ^ 2) ^ {- 1} = 1 + t ^ 2 + (t ^ 2) ^ 2 + (t ^ 2 ^) 3+ \ cdots $.Cela ne peut se produire que si $ | t ^ 2 |<1, $ qui est satisfait
À propos des moments impairs: avez-vous tracé la distribution?Quel serait le moyen?le biais?pouvez-vous déduire quelque chose à propos des moments impairs pour les distributions symétriques?
J'ai tracé la distribution.La moyenne semble être 0 et le graphique est symétrique.J'ai trouvé ce lien qui déduit que les moments impairs de distributions symétriques sont 0. Comment pourrais-je montrer ce résultat en utilisant mgfs?https://math.stackexchange.com/questions/72451/odd-order-moments-of-a-symmetrical-distribution
Vous n'avez pas de texte ou de théorie pour vous soutenir dans vos exercices?Je crois que vous devriez vraiment explorer cette question vous-même en premier.Commencez peut-être ici https://stats.stackexchange.com/questions/238776/
Vous avez $ f x (x) $ où vous avez besoin de $ f X (x).\ qquad $
Le texte que j'utilise est Casella & Berger.
Il ne contient pas l'explication de ce que vous êtes censé faire avec la fonction génératrice de moment pour obtenir les moments?
Il ne donne que la propriété suivante: $ E (X ^ n) = M_X ^ {(n)} (0) = \ frac {d ^ n} {dt ^ n} M_X (t) | _ {t = 0} $.Donc, j'aurais $ E (X ^ {2n}) = M_X ^ {(2n)} (0) = \ frac {d ^ {2n}} {dt ^ {2n}} M_X (t) | _ {t= 0} $, que je ne sais pas généraliser à une formule.
J'ai donc calculé les première, deuxième, troisième et quatrième dérivées.J'ai eu $ E (X ^ 1) = 0, E (X ^ 2) = 2, E (X ^ 3) = 0, $ et $ E (X ^ 4) = 12 $.Ces dérivées sont assez longues à calculer à ce stade, donc je me demande s'il existe un moyen plus simple de procéder pour obtenir une formule pour les evens.Avec les deuxième et quatrième dérivées, la formule de $ E (X ^ {2n}) $ semble encore floue.
Cinq réponses:
probabilityislogic
2019-09-17 06:13:19 UTC
view on stackexchange narkive permalink

Une façon de faciliter votre approche mgf du problème est d'utiliser la série Power

$$ (1-t ^ 2) ^ {- 1} = \ sum_ {j = 0} ^ {\ infty} t ^ {2j} $$ span >

Différencier les rhs à plusieurs reprises est beaucoup plus facile que de différencier les lhs. (notez que cela ne s'applique que pour $ | t | <1 $ mais comme vous vous différenciez à $ t = 0 $ span > ça marche toujours). Vous devriez voir que ce ne sera qu'une factorielle. Cependant, est-il probablement encore plus simple d'évaluer directement l'attente

$$ E (X ^ {2n}) = \ int _ {- \ infty} ^ {\ infty} \ frac {x ^ {2n}} {2} \ exp (- | x |) dx $$

Le résultat doit être une fonction gamma (ie factorielle). Cela ressort également de la série mgf power.

update concernant l'approche mgf, pour évaluer $ E (X ^ {2n}) $ nous devons différencier les rhs $ 2n $ fois. Le résultat suivant sera utile

$$ \ frac {\ partial ^ k x ^ {r}} {\ partial x ^ k} = \ left ( \ begin {matrice} \ frac {r!} {(r-k)!} x ^ {r-k} & r = k, k + 1, \ dots \\ 0 & r = 1, \ points, k-1 \ end {matrice} \droite) $$

Maintenant, si vous appliquez ceci au terme $ t ^ {2j} $ et prenez le dérivé "2n-th", nous avons $ r = 2j $ et $ k = 2n $ et $ x = t $ . Ensuite, nous obtenons $$ \ frac {\ partial ^ {2n} t ^ {2j}} {\ partial t ^ {2n}} = \ left ( \ begin {matrice} \ frac {(2j)!} {(2j-2n)!} t ^ {2j-2n} & 2j = 2n, 2n + 2,2n + 4,2n + 6, \ points \\ 0 & 2j = 0,2, \ points, 2n-2 \ end {matrice} \droite) $$

Cela signifie que lorsque nous additionnons tous les termes, nous pouvons écrire ceci comme $$ \ sum_ {j = 0} ^ {\ infty} \ frac {\ partial ^ {2n} t ^ {2j}} {\ partial t ^ {2n}} = 0 + \ points + 0 + (2n)! + t ^ 2 \ frac {(2n + 2)!} {2!} + t ^ 4 \ frac {(2n + 4)!} {4!} + \ points $ $

seul le tertm unique $ (2n)! $ n'est pas $ 0 $ et pas non plus un multiplesur $ t $ .Ainsi, la définition de $ t = 0 $ ne laisse que ce terme.

Merci de fournir les deux méthodes pour résoudre ce problème.Je pense que j'utiliserais cette dernière approche si on me donnait le choix - cela me semble plus direct.
Ma deuxième dérivée des rhs est $ \ Sigma ^ \ infty _ {j = 0} [2j (2j-1) t ^ {2j-2}] $, mais quand cela est évalué à $ t = 0 $, c'estégal à zéro.Où suis-je en train de foirer?
Il y aura toujours un terme qui a une puissance nulle, de sorte que lorsque $ t = 0 $ vous obtiendrez $ t ^ 0 = 0 ^ 0 = 1 $, et le résultat sera le coefficient de cette puissance.
Je vois. Je vous remercie.
Les mises à jour de cet article sont très appréciées.Merci pour l'explication.
Ben
2019-09-17 03:45:43 UTC
view on stackexchange narkive permalink

Hint: Voici un exemple de fonction de densité de probabilité qui est symétrique autour de zéro:

$$ f_X (0 + x) = f_X (0-x) \ quad \ quad \ quad \ text {pour tous} x \ in \ mathbb {R}. $$

Visuellement, cela signifie que la distribution se reflète autour de la ligne zéro et est la même des deux côtés.Voyez si vous pouvez utiliser cette propriété pour déterminer (puis prouver) quels seraient les moments impairs.Les moments pairs sont un peu plus compliqués, mais voyez si vous pouvez utiliser cette propriété pour réduire ces moments à une forme plus simple.

Merci pour cet indice.Je vois comment des moments impairs seraient égaux à 0 autour d'une distribution symétrique, mais comment puis-je obtenir ce résultat à partir de mon MGF?
Je pense que la meilleure méthode est de l'étendre en tant que série de puissance, comme le montre l'excellente réponse de [Probabilityislogic] (https://stats.stackexchange.com/users/2392/probabilityislogic).
Dilip Sarwate
2019-09-17 07:06:29 UTC
view on stackexchange narkive permalink

Étant donné que l'OP semble avoir des difficultés avec les différents indices dans les commentaires et les autres réponses, voici une méthode heuristique qui donne la bonne réponse dans ce cas. \ begin {align} E [\ exp (tX)] & = E \ left [1 + tX + \ frac {(tX) ^ 2} {2!} + \ Frac {(tX) ^ 3} {3!} + \ Cdots \ right ] \\ & = 1 + tE [X] + \ frac {t ^ 2} {2!} E [X ^ 2] + \ frac {t ^ 3} {3!} E [X ^ 3] + \ cdots \ tag { 1}\\ \ end {align} et donc si nous avons une série de puissance pour $ E [\ exp (tX)] = M_X (t) $ que nous avons réussi à trouver par crochet ou par escroc et sans regarder l'équation affichée ci-dessus car cela ne servira qu'à nous confondre, alors nous pouvons simplement regarder le coefficient de $ t ^ n $ dans la série de puissance que nous avons et disons "Hey Ma! Je pense que le coefficient de $ t ^ n $ est juste $ \ frac {1} {n!} E [X ^ n] $ et je peux donc trouver $ E [X ^ n] $ en multipliant le coefficient que j'ai déjà par $ n! $ .

Pour le cas spécifique de l'OP, il a déjà trouvé que $$ M_X (t) = \ frac {1} {1-t ^ 2} = 1 + t ^ 2 + t ^ 4 + \ cdots + t ^ {2n} + \ cdots \ tag {2} $$ (c'est la partie "par crochet ou par escroc") et ainsi il peut comparer $ (1) $ et $ (2 ) $ pour découvrir quels sont les moments. Je laisserai à l'OP le soin de dire à Ma que $ E [X ^ n] = 0 $ chaque fois que $ n $ span> est étrange.S'il souhaite dire à Ma ce que vaut $ E [X ^ n] $ pour même $ n $ c'est à lui de décider.Le $ E [X ^ 4] = 12 $ que l'OP prétend (dans un commentaire) avoir calculé ne me semble pas correct (je pense qu'il devrait l'être $ E [X ^ 4] = 4! = 24 $ ), mais qui suis-je pour m'immiscer dans la relation sacrée entre la mère et l'enfant?

Merci pour votre contribution divertissante.J'entends ce que vous dites.Donc j'obtiens que $ E [X ^ n] = n! $, Puisque nous multiplions $ n! $ Par le coefficient de chaque $ t ^ n $, où $ n $ est pair.Maintenant, je comprends comment vous avez développé à la fois $ E [e ^ {tX}] $ et $ M_X (t) $, mais comment pouvez-vous utiliser les coefficients des deux extensions l'un avec l'autre?J'aimerais avoir une vue d'ensemble.
whuber
2019-09-17 17:42:00 UTC
view on stackexchange narkive permalink

Si vous ne souhaitez pas faire de calculs inutiles, it est pratique pour voir votre distribution comme un mélange égal d'un exponentiel et de son négatif:

$$ \ frac {1} {2} e ^ {- | x |} = \ frac {1} {2} e ^ {- x} \, \ mathcal {I} (x \ gt 0) + \ frac {1} {2} e ^ {x} \, \ mathcal {I} (x \ lt 0). $$

Parce que $ ((- 1) ^ n + (1) ^ n) / 2 $ est soit $ - 1 + 1 = 0 $ ou $ (1 + 1) / 2 = 2/2 = 1 $ comme $ n $ est respectivement impair ou pair, les moments impairs de votre distribution sont nuls et les moments pairs sont les mêmes que ceux de l'exponentiel. Mais, par définition, le pair Les moments exponentiels sont

$$ \ mu_ {2k} = \ int_0 ^ \ infty x ^ {2k} e ^ {- x} \ mathrm {d} x = \ Gamma (2k + 1 ) = (2k)! $$

Comme cela ne nécessitait aucun calcul plus difficile que $ 1 + 1 = 2, $ , il est difficile d'imaginer une solution plus simple.


Si vous souhaitez plutôt poursuivre l'approche de génération de moment, commence par noter que le mgf de la distribution exponentielle est $$ \ phi (t) = \ int_0 ^ \ infty e ^ {tx} e ^ {- x} \ mathrm {d} x = \ int_0 ^ \ infty e ^ {- (1-t) x} \ mathrm {d} x = \ frac {1} {1-t} . $$ Ainsi, la mgf de ce mélange est

$$ (\ phi (t) + \ phi (-t)) / 2 = \ frac {1} {2} \ left (\ frac {1} {1 -t} + \ frac {1} {1 - (- t)} \ right) = \ frac {1} {1-t ^ 2}. $$

Ceci est analytique près de $ 0 $ et est donc égal à l'expansion des séries de puissance donnée par le théorème binomial comme

$$ \ frac {1} {1-t ^ 2} = (1 + (-t ^ 2)) ^ {- 1} = \ sum_ {k = 0 } ^ \ infty \ binom {-1} {k} (-t ^ 2) ^ k = \ sum_ {k = 0} ^ \ infty t ^ {2k} = \ sum_ {k = 0} ^ \ infty \ color {red} {(2k)!} \, \ color {gray} {\ frac {t ^ {2k}} {(2k)!}}, $$

d'où vous pouvez lire les moments comme les coefficients de $ t ^ n / n!: $ encore une fois, nous voyons qu'ils sont nuls pour les moments impairs et $ (2k)! $ pour les moments pairs.

Merci de partager à la fois la manière la plus simple et la manière mgf.
Sextus Empiricus
2019-09-17 19:00:56 UTC
view on stackexchange narkive permalink

J'ai donc calculé les première, deuxième, troisième et quatrième dérivées. J'ai reçu $ E (X ^ 1) = 0 $ , $ E (X ^ 2) = 2 $ span >, $ E (X ^ 3) = 0 $ et $ E (X ^ 4) = 12 $ span>. Ces dérivées sont assez longues à calculer à ce stade, donc je me demande s'il existe un moyen plus simple de procéder pour obtenir une formule pour les évens.

Vous pouvez utiliser l'extension de la série Taylor:

$$ \ frac {1} {1-t ^ 2} = \ sum_ {k = 0} ^ \ infty t ^ {2k} $$

Cependant, c'est un peu un raisonnement circulaire puisque l'expansion de la série de Taylor est elle-même dérivée du calcul des dérivées. Dans ce cas, vous pouvez tout aussi bien rechercher directement une formule pour les moments d'ordre supérieur de la distribution Laplace.


Vous pourriez trouver indirectement l'expansion de la série de Taylor - sans utiliser $ f (x) = \ sum_ {n = 0} ^ \ infty f ^ {(n)} / n! t ^ k $ - en utilisant à la place la formule pour une série géométrique.

Cependant, vous pouvez également dériver `` manuellement '' les dérivés (ce qui signifie un calcul simple en utilisant une règle de chaîne et une règle de produit) et lorsque vous regardez le modèle des termes, vous constaterez que de nombreux termes deviennent zéro et un un motif régulier émerge.

Disons que nous remplaçons $ u = t ^ 2 $ alors la dérivation semble plus simple:

$$ \ frac {\ text {d} ^ n} {\ text {d} u ^ n} \ frac {1} {(1-u)} = \ frac {n!} {(1-u) ^ n} $$

Maintenant, utilisez la formule de Faà di Bruno (règle de chaîne mais appliquée plusieurs fois):

$$ \ frac {\ text {d} ^ n} {\ text {d} t ^ n} \ frac {1} {(1-u)} = \ somme_ {k = 1} ^ n \ frac {k!} {(1-u) ^ k} \ cdot B_ {n, k} (2t, 2,0, ..., 0) $$

$ B_ {n, k} $ fait référence aux polynômes de Bell. La plupart des termes seront nuls et vous obtenez

$$ \ frac {\ text {d} ^ {2n}} {\ text {d} t ^ {2n}} \ frac {1} {(1-t^ 2)} = \ sum_ {k = 0} ^ n c_ {nk} \ frac {t ^ {2k}} {(1-t ^ 2) ^ {1 + n + k}} $$

avec

$$ c_ {nk} = 2 ^ {2k} \ frac {(2n)!\ cdot (n + k)!} {(n-k)!\ cdot (2k)!} $$

et pour la valeur à $ t = 0 $ que vous avez

$$ \ frac {\ text {d} ^ {2n}} {\ text {d} t ^ {2n}} \ frac {1} {(1-t^ 2)} = c_ {n0} = (2n)!$$

Bien que je ne connaisse pas la formule de Faa di Bruno, j'aime la structure de votre réponse.Merci pour votre ajout!
Lorsque vous différenciez une fois, vous obtenez $ 2t \ cdot \ frac {1} {(1-t ^ 2)} $ deux fois $ 2 \ cdot \ frac {1} {(1-t ^ 2) ^ 2} + 2t \cdot4t \ cdot \ frac {1} {(1-t ^ 2) ^ 3} $ et ainsi de suite.Ce sont des combinaisons de dérivés d'ordre supérieur de $ f (u) = 1 / (1-u) $ et $ g (t) $ qui apparaissent en raison de la différenciation répétée (application de règles de chaîne et de produit créant ces multiples termes).La formule de Faa di Bruno tient compte de ces termes.Dans ce cas, comme les dérivées de $ t ^ 2 $ se terminent à un moment donné, il devient relativement facile de garder une trace des termes.Bien sûr, d'autres façons de dériver la série sont plus faciles, mais celle-ci est également cool.
C'est un résultat vraiment intéressant.Merci de l'avoir partagé.
C'est en fait un moyen fastidieux de calculer $! N ((1-t) ^ {- n} - (-1-t) ^ {- n}) $


Ce Q&R a été automatiquement traduit de la langue anglaise.Le contenu original est disponible sur stackexchange, que nous remercions pour la licence cc by-sa 4.0 sous laquelle il est distribué.
Loading...